1. Trang chủ
  2. » Luận Văn - Báo Cáo

(Luận văn) tính bất khả quy của đa thức với hệ số nguyên

41 3 0

Đang tải... (xem toàn văn)

Tài liệu hạn chế xem trước, để xem đầy đủ mời bạn chọn Tải xuống

THÔNG TIN TÀI LIỆU

ĐẠI HỌC THÁI NGUYÊN TRƯỜNG ĐẠI HỌC KHOA HỌC  - PHẠM THỊ THU TRANG lu an n va to ie gh tn TÍNH BẤT KHẢ QUY p CỦA ĐA THỨC VỚI HỆ SỐ NGUYÊN d oa nl w ll u nf va an lu oi m LUẬN VĂN THẠC SĨ TOÁN HỌC z at nh z m co l gm @ an Lu THÁI NGUYÊN - 2019 n va ac th si ĐẠI HỌC THÁI NGUYÊN TRƯỜNG ĐẠI HỌC KHOA HỌC  - PHẠM THỊ THU TRANG lu an va n TÍNH BẤT KHẢ QUY p ie gh tn to CỦA ĐA THỨC VỚI HỆ SỐ NGUYÊN oa nl w Chuyên ngành: Phương pháp Toán sơ cấp d Mã số: 46 01 13 ll u nf va an lu oi m LUẬN VĂN THẠC SĨ TOÁN HỌC z at nh NGƯỜI HƯỚNG DẪN KHOA HỌC z GS.TS Lê Thị Thanh Nhàn m co l gm @ an Lu THÁI NGUYÊN - 2019 n va ac th si Mục lục Lời cảm ơn Mở đầu lu an n va p ie gh tn to Tiêu chuẩn Eisenstein tiêu chuẩn rút gọn theo module số nguyên tố 1.1 Tiêu chuẩn Eisenstein số mở rộng 1.2 Tiêu chuẩn rút gọn theo module số nguyên tố toán ngược 11 trị khả nghịch, giá trị nguyên tố tính bất khả Giá trị khả nghịch tính bất khả quy Giá trị nguyên tố tính bất khả quy Một tiêu chuẩn tính bất khả quy Giá trị nguyên tố đối số đủ lớn tính bất khả quy d nf va an lu 16 16 21 27 32 37 z at nh oi Tài liệu tham khảo quy lm ul Kết luận oa nl w Giá 2.1 2.2 2.3 2.4 38 z m co l gm @ an Lu n va ac th si Lời cảm ơn lu an n va p ie gh tn to Trước tiên xin gửi lời cảm ơn chân thành sâu sắc tới GS.TS Lê Thị Thanh Nhàn Mặc dù bận rộn công việc, song từ ngày Cơ ln tận tình bảo, hướng dẫn đưa lời khun có ích giúp tơi hồn thiện luận văn Tôi xin gửi lời cảm ơn tới thầy, cán khoa Tốn - Tin, trường Đại học Khoa học - Đại học Thái Nguyên, Ban giám hiệu đồng nghiệp trường Trung học phổ thơng Hồnh Bồ - Tỉnh Quảng Ninh bạn tập thể lớp Cao học Toán K11D, không trang bị cho kiến thức bổ ích mà cịn ln ln giúp đỡ tơi, tạo điều kiện cho thời gian theo học trường Cuối cùng, tơi xin chân thành bày tỏ lịng biết ơn đến gia đình, bạn bè, người khơng ngừng ủng hộ, động viên, hỗ trợ tạo điều kiện giúp tơi vượt qua khó khăn để hoàn thiện luận văn d oa nl w nf va an lu z at nh oi lm ul z m co l gm @ an Lu n va ac th si Mở đầu lu an n va p ie gh tn to Tính bất khả quy đa thức với hệ số nguyên trường số phức C trường số thực R giải từ kỷ 19 thông qua Định lý Đại số Tuy nhiên, tính bất khả quy đa thức với hệ số nguyên trường số hữu tỷ Q đến thách thức nhà Toán học giới Trong luận văn này, tác giả trình bày lại số tiêu chuẩn bất khả quy đa thức trường số hữu tỷ Q với hệ số nguyên báo gần [8] [11] Luận văn gồm chương Trong chương 1, chúng tơi trình bày hai tiêu chuẩn bất khả quy tiếng Phần 1.1 trình bày Tiêu chuẩn Eisenstein mở rộng Phần 1.2 trình bày tiêu chuẩn rút gọn theo module số nguyên tố phát biểu đảo tiêu chuẩn Nội dung chương viết theo báo [11] cuả R Thangadurai năm 2007 Chương trình bày tiêu chuẩn bất khả quy trường số hữu tỷ Q liên quan đến giá trị khả nghịch giá trị nguyên tố đa thức với hệ số nguyên Phần 2.1 trình bày tiêu chuẩn liên quan giá trị khả nghịch với tính bất khả quy đa thức Phần 2.2 trình bày mối quan hệ giá trị nguyên tố tính bất khả quy Các kết hai phần viết dựa theo báo [11] R Thangadurai năm 2007 Phần 2.3 trình bày tiêu chuẩn bất khả quy trường Q số hữu tỷ liên quan đến đa thức có hệ số nguyên tăng dần theo số có hệ số cao nguyên tố nhận giá trị nguyên tố Kết phần viết dựa theo báo [8] A Jakhar N Sangwan năm 2018 Phần 2.4 trình bày giá trị nguyên tố đối số đủ lớn tính bất khả quy đa thức với hệ số nguyên Nội dung phần viết sở nội dung báo [11] R Thangadurai năm 2007 d oa nl w nf va an lu z at nh oi lm ul z m co l gm @ an Lu n va ac th si lu Trong luận văn này, tiêu chuẩn phần 2.1 giá trị khả nghịch tính bất khả quy; phần 2.2 giá trị nguyên tố tính bất khả quy; phần 2.3 tiêu chuẩn cho tính bất khả quy kết chưa trình bày luận văn thạc sĩ trước Hơn thế, phần 1.1, 1.2, 2.4, có số kết quen biết trình bày vài luận văn trước (xem [1], [2]), cách chứng minh ví du mới, tác giả luận văn tự tính tốn Đặc biệt luận văn [2], Nguyễn Văn Lập chứng minh đa thức x4 − 2x2 + bất khả quy Q không bất khả quy Zp với số nguyên tố p cách sử dụng kiến thức nhóm, luận văn chứng minh đa thức x4 + bất khả quy Q khả quy Zp với số nguyên tố p cách sử dụng kiến thức trường hữu hạn an n va p ie gh tn to Thái Nguyên, ngày 25 tháng năm 2019 Tác giả luận văn nl w d oa Phạm Thị Thu Trang nf va an lu z at nh oi lm ul z m co l gm @ an Lu n va ac th si Chương Tiêu chuẩn Eisenstein tiêu chuẩn rút gọn theo module số nguyên lu an tố n va tn to p ie gh Một đa thức với hệ số trường gọi bất khả quy có bậc dương khơng phân tích thành tích hai đa thức có bậc thấp Một đa thức bậc dương với hệ số trường khả quy tích hai đa thức với bậc thấp Chú ý tính bất khả quy đa thức phụ thuộc vào trường sở Chẳng hạn, đa thức x2 − bất khả quy trường Q số hữu tỷ, không bất khả quy trường R số thực Đa thức x2 + bất khả quy trường R không bất khả quy trường C số phức Tính bất khả quy trường số phức trường số thực làm rõ nhờ Định lý Đại số: Mọi đa thức bậc dương với hệ số phức có nghiệm phức Vì đa thức bất khả quy C đa thức bậc Các đa thức bất khả quy R đa thức bậc đa thức bậc hai có biệt thức âm Câu hỏi đặt đa thức f (x) cho khả quy hay bất khả quy Q? Cho đến nay, khơng có điều kiện cần đủ áp dụng cho tất đa thức, mà ta có số tiêu chuẩn để kiểm tra tính bất khả quy số trường hợp cụ thể Rõ ràng đa thức bậc bất khả quy Q Các đa thức bậc hai bậc ba bất khả quy Q khơng có nghiệm d oa nl w nf va an lu z at nh oi lm ul z m co l gm @ an Lu n va ac th si lu an n va 1.1 Tiêu chuẩn Eisenstein số mở rộng p ie gh tn to hữu tỷ Đối với đa thức bậc lớn 3, đa thức có nghiệm hữu tỷ khơng bất khả quy Tuy nhiên điều ngược lại không Chẳng hạn, đa thức (x2 + 1)2 khơng có nghiệm hữu tỷ, khơng bất khả quy Trong chương này, chúng tơi trình bày hai tiêu chuẩn tiếng tính bất khả quy trường số hữu tỷ Q đa thức với hệ số nguyên dựa theo báo [11] R Thangadurai Phần thứ dành để trình bày Tiêu chuẩn Eisensrein số mở rộng Mở rộng thứ phát H Chao báo A Generalization of Eisenstein’s Criterion, Mathematics Magazine, Vol 47 (1974), 158-159 mở rộng thứ hai đưa S H Weintraub báo A mild generazation of Eisenstein criterion, Proceedings of the American Mathematical Society, Vol 141 (2013), 1159-1160 Phần trình bày tiêu chuẩn bất khả quy phổ biến nhất, tiêu chuẩn rút gọn theo module số nguyên tố Phát biểu đảo tiêu chuẩn khơng cịn nữa, đưa chứng minh chi tiết để minh họa điều w d oa nl Trong mục này, chúng tơi trình bày lại tiêu chuẩn Eisenstein số mở rộng liên quan tính bất khả quy đa thức với hệ số nguyên trường số hữu tỉ Q Đây tiêu chuẩn quen thuộc thường sử dụng làm tốn tính bất khả quy đa thức Q Cho f (x) = an xn + an−1 xn−1 + · · · + a1 x + a0 nf va an lu z at nh oi lm ul đa thức bậc n với ∈ Z, an 6= z Tiêu chuẩn bất khả quy biết đến nhiều tiêu chuẩn Eisenstein, phát biểu sau 1.1.1 Định lý Cho đa thức f (x) = an xn + an−1 xn−1 + · · · + a1 x + a0 đa thức với hệ số nguyên có bậc n > Nếu tồn số nguyên tố p cho p - an , p | với i = 0, 1, , n − p2 - a0 , đa thức f (x) bất khả quy Q m co l gm @ an Lu n va Chứng minh Giả sử f (x) khả quy Q Theo Bổ đề Gauss, tồn biểu ac th si diễn f (x) = g(x)h(x), g(x) = bm xm + · · · + b1 x + b0 ∈ Z[x] h(x) = ck xk + · · · + c1 x + c0 ∈ Z[x] với deg g(x) = m, deg h(x) = k m, k < n Do p ước a0 = b0 c0 nên p | b0 p | c0 Mặt khác, p2 không ước a0 nên hai số b0 c0 , có số chia hết cho p Giả thiết p | c0 Khi b0 khơng chia hết cho p Vì an = bm ck p - an nên bm ck không chia hết cho p Do tồn số r bé cho cr khơng bội p Ta có ar = b0 cr + (b1 cr−1 + b2 cr−2 + · · · + br c0 ) Vì r ≤ k < n nên p | ar Theo cách chọn r ta có lu p | b1 cr−1 + b2 cr−2 + · · · + br c0 an n va Suy p | b0 cr , điều vơ lí hai số b0 cr không bội p Vậy f (x) bất khả quy Q tn to p ie gh Các đa thức thỏa mãn Định lý gọi đa thức Eisenstein Chẳng hạn, đa thức x5 − 4x4 + 18x3 + 24x2 + 4x + đa thức Eisenstein bất khả quy theo Tiêu chuẩn Eisenstein với p = Thông thường, Tiêu chuẩn Eisenstein không áp dụng trực tiếp cho đa thức f (x), mà áp dụng cho đa thức f (x + a) với a số Chú ý đa thức f (x) bất khả quy Q đa thức f (x + a) bất khả quy Q với số nguyên a Do vậy, cố gắng tìm số a với hy vọng biến đổi đa thức f (x + a) ta đa thức thỏa mãn điều kiện Tiêu chuẩn Eisenstein Dưới ví dụ tính bất khả quy đa thức chia đường tròn thứ p với p số nguyên tố 1.1.2 Ví dụ Cho p số nguyên tố Khi đa thức chia đường trịn thứ p f (x) = xp−1 + xp−2 + · · · + x + d oa nl w nf va an lu z at nh oi lm ul z @ gm bất khả quy Q m co l Chứng minh Đa thức f (x) = xp−1 + xp−2 + · · · + x + có hệ số nên khơng thể áp dụng trực tiếp Tiêu chuẩn Eisenstein để xét tính bất khả quy f (x) an Lu n va ac th si xp − Chú ý f (x) = Suy ra, chọn a = ta có x−1 (x + 1)p − = xp−1 + Cp1 xp−2 + + Cpp−2 x + Cpp−1 , f (x + 1) = x p! Cpk = số tổ hợp chập k p phần tử Do p nguyên (p − k)!k! tố nên Cpk bội p với k = 1, 2, , p − Cpp−1 = p không bội p2 Vì f (x + 1) bất khả quy theo Tiêu chuẩn Eisenstein (áp dụng cho số nguyên tố p) Do f (x) bất khả quy Q lu an n va p ie gh tn to Như vậy, thơng qua tiêu chuẩn Eisenstein, từ tốn ban đầu xét tính bất khả quy đa thức bậc n với hệ số nguyên, ta đưa tốn phân tích n hệ số đa thức f (x + a), sau biến đổi đa thức f (x + a) cần tìm ước chung nguyên tố phù hợp hệ số, trừ hệ số cao nhất, đa thức f (x + a) Hiển nhiên, cố gắng biến đổi đa thức để tạo đa thức với hệ số lớn hơn, nhiệm vụ sau tính tốn kiểm tra ước nguyên tố chung hệ số thỏa mãn điều kiện Tiêu chuẩn Eisenstein Tuy nhiên, chưa chắn tồn phép biến đổi để đa thức ban đầu chuyển thành đa thức áp dụng tiêu chuẩn Eisenstein, tức chưa tìm số nguyên a để đa thức f (x + a) áp dụng Tiêu chuẩn Eisenstein ứng với số nguyên tố p Ví dụ, người ta đa thức x4 − 10x2 + bất khả quy Q khơng tìm số ngun a để đa thức (x + a)4 − 10(x + a)2 + d oa nl w nf va an lu lm ul z at nh oi bất khả quy theo Tiêu chuẩn Eisenstein với số nguyên tố p Trong phần cuối mục này, nhắc lại số mở rộng Tiêu chuẩn Eisenstein Trước hết nhắc lại tiêu chuẩn bất khả quy H Chao báo A Generalization of Eisenstein’s Criterion, Mathematics Magazine, Vol 47 (1974), 158-159 1.1.3 Định lý Cho f (x) = an xn + + a1 x + a0 đa thức bậc n với hệ số nguyên Giả sử p số nguyên tố cho có hai số t 6= k thỏa mãn: p không ước at , p ước với i 6= t p2 không ước ak Khi f (x) tích hai đa thức với hệ số nguyên, hai đa thức có bậc lớn | t − k | z m co l gm @ an Lu n va ac th si Mặt khác deg f (x) = tồn bảy số m1 = −22, m2 = −8, m3 = −4, m4 = 0, m5 = 12, m6 = 18 m7 = 30 thỏa mãn tính chất hiệu hai số có giá trị tuyệt đối lớn Hơn giá trị f (x) bảy số số nguyên tố, số đối số nguyên tố khả nghịch Theo Định lý 8, đa thức f (x) bất khả quy Q lu an n va p ie gh tn to Chú ý tính bất khả quy đa thức f (x) Ví dụ suy từ tiêu chuẩn trình bày phần trước: Tiêu chuẩn Eisenstein mở rộng; tiêu chuẩn rút gọn theo module số nguyên tố; tiêu chuẩn bất khả quy liên quan đến số lần đa thức nhận giá trị khả nghịch (Định lý 6) Các Định lý Định lý 10 trình bày phát biểu dựa nội dung báo Irreducibility of Polynomials Whose Coefficients are Integers R Thangadurai tạp chí Mathematics Newsletter, 17, (2007), 29-37 2.2.6 Định lý Cho f (x) đa thức bậc n > với hệ số nguyên Nếu P (f ) + 2u(f ) ≥ n + f (x) bất khả quy trường số hữu tỷ Q d oa nl w Chứng minh Ta chứng minh phương pháp phản chứng Giả sử đa thức f (x) không bất khả quy Q Theo Bổ đề Gauss, f (x) có phân tích f (x) = g(x)h(x), g(x) h(x) đa thức có bậc dương với hệ số ngun Khơng làm tính tổng qt, giả sử `(g) ≥ `(h), `(g) = u(g) − deg g(x), `(h) = u(h) − deg h(x) Định nghĩa Ta chứng minh nf va an lu lm ul `(g) + `(h) ≥ P (f ) + 2u(f ) − n z at nh oi Giả sử m ∈ Z cho f (m) số nguyên tố Ta có f (m) = g(m)h(m) Suy g(m) h(m) nhận giá trị −1 Trong với m ∈ Z cho f (m) khả nghịch ta có g(m) h(m) nhận giá trị khả nghịch Do u(g) + u(h) ≥ P (f ) + 2u(f ) z l gm @ Ta có co m `(g) + `(h) = u(g) − deg g(x) + u(h) − deg h(x) = u(g) + u(h) − n an Lu ≥ P (f ) + 2u(f ) − n n va 25 ac th si Theo giả thiết P (f ) + 2u(f ) ≥ n + 4, suy P (f ) + 2u(f ) − n ≥ Do vậy, ta `(g) + `(h) ≥ Nếu `(g) > `(h) > theo Định nghĩa 2, ta có đa thức g(x), h(x) đa thức béo Theo Mệnh đề , ta có deg g(x), deg h(x) ≤ kéo theo tổng chúng ≥ Do đó, có g(x) h(x) đa thức béo Khơng làm tính tổng qt, giả sử g(x) đa thức béo Vì h(x) khơng đa thức béo n ≥ nên deg h(x) ≥ `(h) ≤ Khơng thế, `(g) + `(h) ≥ 4, ta có `(g) = u(g) − deg g(x) ≥ suy u(g) ≥ + deg g(x), mâu thuẫn deg g(x) ≤ u(g) ≤ deg g(x) Do f (x) phải bất khả quy trường số hữu tỷ Q lu 2.2.7 Ví dụ Đa thức f (x) = x4 − 5x2 + đa thức bất khả quy trường số hữu tỷ Q an n va p ie gh tn to Chứng minh Ta có deg f (x) = f (x)+1 = (x−1)(x+1)(x−2)(x+2) Vì f (x) nhận giá trị −1 điểm Vì f (4) = f (−4) = 179 số nguyên tố, nên f (x) nhận giá trị nguyên tố lần Như ta có nl w P (f ) + 2u(f ) ≥ + 2.4 = 10 > deg f (x) + d oa Vậy đa thức f (x) bất khả quy Q theo Định lý 2.2.6 nf va an lu 2.2.8 Định lý 10 Cho f (x) đa thức với hệ số nguyên có bậc n ≥ Nếu P (f ) ≥ n + f (x) bất khả quy trường số hữu tỷ Q lm ul Chứng minh Nếu P (f ) ≥ n + rõ ràng z at nh oi P (f ) + 2u(f ) ≥ n + z Trong trường hợp này, theo Định lý 9, đa thức f (x) bất khả quy Vì vậy, ta cần chứng minh định lý trường hợp P (f ) = n + Giả sử điều ngược lại, tức f (x) không bất khả quy Khi theo Bổ đề Gauss, ta viết f (x) = g(x)h(x) g(x), h(x) đa thức với hệ số nguyên có bậc dương Theo chứng minh Định lý ta có co l gm @ m `(g) + `(h) ≥ P (f ) + 2u(f ) − n, an Lu `(g) `(h) xác định Định nghĩa n va 26 ac th si Vì P (f ) = n + nên `(g) + `(h) ≥ n + + 2u(f ) − n = + 2u(f ) Do `(g) `(h) phải dương Vì deg f (x) ≥ theo giả thiết, nên theo Mệnh đề 7, g(x) h(x) phải đa thức béo Không tổng quát, giả sử g(x) đa thức béo Khi đó, h(x) khơng đa thức béo Vì `(g) ≥ `(h) ≤ Suy `(g) + `(h) ≤ `(g) Tuy nhiên ta có `(g) + `(h) ≥ P (f ) + 2u(f ) − n ≥ n + − n = lu Do `(g) ≥ Suy u(g) ≥ deg g(x) + Tuy nhiên theo lập luận phần cuối chứng minh Định lý ta suy u(g) ≤ deg g(x) + 1, mâu thuẫn với u(g) ≥ deg g(x) + Do đa thức f (x) bất khả quy Q an n va p ie gh tn to 2.2.9 Ví dụ 10 Đa thức f (x) = x7 − 14x5 + 49x3 − 36x + 11 bất khả quy Q w Chứng minh Ta có n = deg f (x) = oa nl f (0) = 11, f (1) = 11, f (−1) = 11, f (2) = 11, f (−2) = 11, f (3) = 11, f (−3) = 11, f (4) = 5051, f (7) = 604811, f (17) = 390700811 d lu nf va an Trong 11, 5051, 604811, 390700811 số nguyên số Như f (x) nhận giá trị nguyên tố 10 lần, hay P (f ) ≥ 10 = n + Do P (f ) ≥ n + Theo Định lý 10, đa thức f (x) = x7 − 14x5 + 49x3 − 36x + 11 bất khả quy Q z at nh oi lm ul 2.3 Một tiêu chuẩn tính bất khả quy z @ m co l gm Mục tiêu phần trình bày tiêu chuẩn tính bất khả quy trường Q đa thức với hệ số nguyên cho hệ số tăng dần theo bậc nhận giá trị nguyên tố Các kết phần viết chủ yếu dựa theo báo A Jakhar N Sangwan năm 2018: An irreducibility criterion for integer polynomials, Amer Math Monthly, 125, 464-465 Kết phần 2.3 Định lý 11 an Lu n va 27 ac th si Gọi C tập điểm phía hình trịn phức bán kính đơn vị, tức C := {z ∈ C : |z| < 1}, lu C trường số phức Trước phát biểu kết mục này, cần số kết bổ trợ sau Chứng minh mệnh đề sau trình bày tài liệu “A Class of Irreducible Polynomials” tác giả J Harrington, L Jones [Mệnh đề 2.3, trang 113-119] xuất năm 2013 2.3.1 Mệnh đề Cho f (x) = an xn + · · · + a0 ∈ Q[x] giả sử 6= 0, aj 6= với ≤ i < j ≤ n Giả sử X |al | ≤ q t |at | an va 0≤l≤n; l6=t n p ie gh tn to ≤ t ≤ n, t 6= i, t 6= j q ∈ R với < q ≤ Nếu f (x) có nghiệm α với α ∈ {z ∈ C | q ≤ |z| ≤ 1} d oa nl w bất đẳng thức trở thành đẳng thức α2(j−i) = Bây chứng minh hai mệnh đề sở mà kết chúng độc lập với thú vị 2.3.2 Mệnh đề Cho đa thức f (x) = an xn + · · · + a0 ∈ Z[x] cho an lu nf va < a0 ≤ a1 ≤ ≤ ak−1 < ak < ak+1 ≤ ≤ an−1 ≤ an z at nh oi lm ul với số k thỏa mãn ≤ k ≤ n − Khi tất nghiệm f (x) thuộc tập C z Chứng minh Đầu tiên ta chứng minh f (x) có tất nghiệm thuộc tập {z ∈ C : |z| ≤ 1} @ co l gm Giả sử ngược lại, f (x) có nghiệm α cho |α| > Khi α nghiệm m F (x) = (x − 1)f (x) = an xn+1 + (an−1 − an )xn + · · · + (a0 − a1 )x − a0 an Lu 28 n va Chú ý module tích hai số phức tích hai module hai số phức Hơn nữa, module tổng hai số phức nhỏ ac th si tổng hai module hai số phức Vì theo giả thiết Mệnh đề giả thiết |α| > ta có |an αn+1 | ≤ a0 + (a1 − a0 )|α| + · · · + (an − an−1 )|α|n < a0 |α|n + (a1 − a0 )|α|n + · · · + (an − an−1 )|α|n = |an αn |, lu điều mâu thuẫn an > Do tất nghiệm f (x) phải thuộc hình trịn bán kính đơn vị Để chứng minh nghiệm α f (x) thuộc C , ta |α| 6= Giả sử |α| = Hiển nhiên hệ số xk xk+1 F (x) âm, ngoại trừ hệ số an dương Do vậy, giả thiết Mệnh đề thỏa mãn với t = n + 1, i = k, j = k + q = Cũng theo Mệnh đề 8, ta có α2 = 1, khơng thể xảy dễ thấy f (1) f (−1) khác theo giả thiết ban đầu Vậy tất nghiệm f (x) thuộc C an n va p ie gh tn to 2.3.3 Mệnh đề 10 Cho đa thức f (x) ∈ Z[x] đa thức có tất nghiệm nằm tập C Nếu tồn số nguyên m với |m| ≥ cho |f (m)| số nguyên tố, f (x) bất khả quy Q d oa nl w Chứng minh Giả sử ngược lại, đa thức f (x) không bất khả quy Q Khi đó, theo Bổ đề Gauss, ta có phân tích f (x) = g(x)h(x), g(x), h(x) ∈ Z[x] Theo giả thiết, |f (m)| số nguyên tố nên |g(m)| |h(m)| Không tổng quát, giả sử |g(m)| = Giả sử deg g(x) = k Theo Định lý Đại số, g(x) phải có k nghiệm C, nghiệm tính với số bội Gọi α1 , , αk nghiệm g(x) Khi chúng nghiệm f (x) nf va an lu lm ul z at nh oi k Y g(x) = c (x − αi ), i=1 z c hệ số cao g(x) Theo giả thiết nghiệm f (x) ta có αi ∈ C, |αi | < với ≤ i ≤ k Theo giả thiết, |m| ≥ Chú ý module hiệu hai số phức lớn hiệu hai module hai số phức Vì ta có co l gm @ m k k k Y Y Y |g(m)| = |c (m − αi )| ≥ |c| (|m| − |αi |) > |c| (|m| − 1) ≥ 1, i=1 an Lu i=1 i=1 n 29 va suy |g(m)| > 1, điều mâu thuẫn ac th si Bằng lập luận tương tự chứng minh Mệnh đề 10, ta thu kết sau 2.3.4 Mệnh đề 11 Cho đa thức f (x) ∈ Z[x] có tất nghiệm tập {z ∈ C : |z| > 1} Nếu |f (0)| số nguyên tố f (x) bất khả quy Q Bây chứng minh kết phần này, nói tiêu chuẩn cho tính bất khả quy Q đa thức với hệ số nguyên thỏa mãn tính chất: hệ số tăng dần theo bậc đa thức nhận giá trị nguyên tố 2.3.5 Định lý 11 Cho đa thức f (x) = an xn + an−1 xn−1 + · · · + a1 x + a0 với hệ số nguyên thỏa mãn điều kiện: lu an (i) ≤ a0 ≤ a1 ≤ ≤ ak−1 < ak < ak+1 ≤ ≤ an với giá trị số k thỏa mãn ≤ k ≤ n − 1; n va tn to (ii) |an | > |an−1 | + · · · + |a0 | với a0 6= p ie gh Giả sử |an | số nguyên tố |f (m)| số nguyên tố với số nguyên m thỏa mãn |m| ≥ Khi f (x) bất khả quy Q Hơn nữa, |m| = ar với a r hai số tự nhiên f (xr ) bất khả quy Q nl w d oa Chứng minh Chú ý đa thức f (x) thỏa mãn điều kiện (i) định lý, theo Mệnh đề 9, tất nghiệm f (x) nằm phía đường tròn đơn vị, tức nằm tập C = {z ∈ C : |z| < 1} Hiển nhiên điều kiện (ii) thỏa mãn |α| ≥ từ tính chất module hiệu hai số phức, ta suy   n−1 n−1 X X |f (α)| ≥ |an ||α|n − |aj ||α|j ≥ |α|n |an | − |aj | > nf va an lu z at nh oi lm ul j=0 j=0 z Vì f (α) 6= Do vậy, trường hợp f (x) thỏa mãn điều kiện (ii) định lý tất nghiệm f (x) phải nằm C Theo  giả thiết nằm a0 6= 0, nên tất nghiệm đa thức g(x) := xn f x tập {z ∈ C : |z| > 1} Do |an | = |g(0)| số nguyên tố theo Mệnh đề 2.3.4, g(x) bất khả quy Q Suy f (x) bất khả quy Q Nếu tồn số nguyên m với |m| ≥ cho |f (m)| nguyên tố f (x) bất khả quy Q theo Mệnh đề 10 Ta phải chứng m co l gm @ an Lu n va 30 ac th si minh |m| lũy thừa bậc r số nguyên, f (xr ) bất khả quy Q Vì tất nghiệm f (x) nằm tập C , tức có module nhỏ 1, nên β nghiệm f (xr ), β r phải nghiệm f (x), tức β bậc r α với α nghiệm f (x) Suy ra, module β phải nhỏ Vì vậy, tất nghiệm f (xr ) nằm C Tương tự, tất nghiệm f (−xr ) nằm C Vì |f (m)| số nguyên tố, nên theo Mệnh đề 10 ta thu kết mong muốn n 2.3.6 Hệ Giả sử 22 + số nguyên tố Khi đa thức n f (x) = (22 + 1)xn + an−1 xn−1 + · · · + a1 x + a0 , lu an với n n va ≤ a0 ≤ ≤ ak−1 < ak < ak+1 ≤ ≤ an−1 ≤ 22 + tn to ≤ k ≤ n − bất khả quy Q n ie gh Chứng minh Ta có 22 + số nguyên tố Khi đa thức p n nl w với f (x) = (22 + 1)xn + an−1 xn−1 + · · · + a1 x + a0 n d oa ≤ a0 ≤ ≤ ak−1 < ak < ak+1 ≤ ≤ an−1 ≤ 22 + nf va an lu ≤ k ≤ n − thỏa mãn điều kiện (i) Định lý 11 Do f (x) đa thức bất khả quy Q z at nh oi lm ul 2.3.7 Ví dụ 11 Đa thức f (x) = 11x5 + 3x4 − 2x3 + x + đa thức bất khả quy Q Chứng minh Đa thức f (x) = 11x5 + 3x4 − 2x3 + x + có a5 = 11, a4 = 3, a3 = −2, a2 = 0, a1 = 1, a0 = Các hệ số thỏa mãn z gm @ |a5 | > |a4 | + |a3 | + |a2 | + |a1 | + |a0 | m co l Hơn nữa, a5 = 11 f (2) = 389 số nguyên tố Như f (x) thỏa mãn điều kiện (ii) Định lý 11 với m = Do f (x) đa thức bất khả quy Q an Lu 31 n va 2.3.8 Ví dụ 12 Đa thức g(x) = 64x5 + 2x4 − 5x3 + 2x2 − bất khả quy trường số hữu tỷ Q ac th si Chứng minh Đa thức g(x) = 64x5 + 2x4 − 5x3 + 2x2 − có a5 = 64, a4 = 2, a3 = −5, a2 = 2, a1 = 0, a0 = −9 Các hệ số thỏa mãn |a5 | > |a4 | + |a3 | + |a2 | + |a1 | + |a0 | Chọn m = 2, ta có g(2) = 64.25 + 2.24 − 5.23 + 2.22 − = 211 − = 2039 số nguyên tố Như g(x) thỏa mãn điều kiện (ii) Định lý 11 với m = Do g(x) đa thức bất khả quy Q lu 2.4 Giá trị nguyên tố đối số đủ lớn tính bất khả quy an n va Chúng ta định nghĩa “chiều cao” cuả f (x) a i H1 = max 0≤i≤n−1 an gh tn to ie p a

Ngày đăng: 24/07/2023, 09:44

Xem thêm:

w